Rate of Change Using Inverse Trig Functions

Click For Summary
The discussion centers on calculating the rate of change of the angle of elevation from a spectator to an elevator ascending at 20 ft/sec. The problem involves using the derivative of arctan to find the angle's increase when the elevator is 50 ft high. One participant used implicit differentiation and confirmed their result of 0.2 rad/sec, while another independently verified the same outcome. There was some confusion about whether the time when the elevator reaches 50 ft was necessary for the calculation, but it was deemed irrelevant. Overall, the participants agreed on the correctness of the solution.
Amrator
Messages
246
Reaction score
83

Homework Statement


A spectator is standing 50 ft from the freight elevator shaft of a building which is under construction. The elevator is ascending at a constant rate of 20 ft/sec. How fast is the angle of elevation of the spectator's line of sight to the elevator increasing when the elevator is 50 ft above the ground?

Homework Equations


Derivative of arctan
g'(x) = 1/[(x^2)+1]

h=height

The Attempt at a Solution


IMG_20150727_192407877.jpg


I don't know how to use latex.

I used implicit differentiation. Since the elevator is moving at a velocity of 20 ft/sec, I plugged that in for dh/dt. Because they wanted me to use the derivative of arctan formula, I'm assuming the input is h/50 (y/x). After I plugged in h/50 into the formula, I plugged in the height they gave me, 50, into h. I multiplied that by 20 (dh/dt) and got 0.2 rad/sec.

Did they want me to find the time when the elevator was 50 ft above the ground first?
Also I apologize for the small writing. It's arctan(h/50).
 
Last edited:
Physics news on Phys.org
I don't think the time when the elevator is 50 ft high is relevant.
 
  • Like
Likes Amrator
Dr. Courtney said:
I don't think the time when the elevator is 50 ft high is relevant.
Am I allowed to ask if the result I got is correct? If not, that's fine.
 
I don't see anything wrong with your work.
 
  • Like
Likes Amrator
Amrator said:
Am I allowed to ask if the result I got is correct? If not, that's fine.
The result looks to be correct.

Working it a somewhat different way, I get the same result.
 
  • Like
Likes Amrator
Alright. Thanks for the help, guys. I really appreciate it.
 
Question: A clock's minute hand has length 4 and its hour hand has length 3. What is the distance between the tips at the moment when it is increasing most rapidly?(Putnam Exam Question) Answer: Making assumption that both the hands moves at constant angular velocities, the answer is ## \sqrt{7} .## But don't you think this assumption is somewhat doubtful and wrong?

Similar threads

  • · Replies 1 ·
Replies
1
Views
3K
  • · Replies 2 ·
Replies
2
Views
2K
Replies
4
Views
2K
  • · Replies 14 ·
Replies
14
Views
4K
  • · Replies 3 ·
Replies
3
Views
2K
  • · Replies 3 ·
Replies
3
Views
2K
  • · Replies 3 ·
Replies
3
Views
2K
Replies
8
Views
2K
  • · Replies 2 ·
Replies
2
Views
4K
  • · Replies 4 ·
Replies
4
Views
5K